Login

Welcome, Guest. Please login or register.

March 29, 2024, 05:21:38 am

Author Topic: VCE Physics Question Thread!  (Read 603325 times)  Share 

0 Members and 4 Guests are viewing this topic.

joey7

  • Victorian
  • Forum Regular
  • **
  • Posts: 74
  • Respect: 0
  • School: Notre Dame College Shepparton
  • School Grad Year: 2013
Re: VCE Physics Question Thread!
« Reply #330 on: November 02, 2013, 04:06:23 pm »
0
What happens to magnetic field (B) when the current increases, and which equation tells us this

Stevensmay

  • Guest
Re: VCE Physics Question Thread!
« Reply #331 on: November 02, 2013, 04:47:36 pm »
+1
What happens to magnetic field (B) when the current increases, and which equation tells us this
Misread question.



So we can see that as the current increases, the magnetic field needs to increase or the time needs to be reduced.
« Last Edit: November 02, 2013, 08:05:34 pm by Stevensmay »

lzxnl

  • Victorian
  • ATAR Notes Legend
  • *******
  • Posts: 3432
  • Respect: +215
Re: VCE Physics Question Thread!
« Reply #332 on: November 02, 2013, 04:52:00 pm »
+2
Magnetic field strength is directly proportional to the current.
What equation tells you that?
Biot-Savart law:

Oh wait, not part of course. My bad.
2012
Mathematical Methods (50) Chinese SL (45~52)

2013
English Language (50) Chemistry (50) Specialist Mathematics (49~54.9) Physics (49) UMEP Physics (96%) ATAR 99.95

2014-2016: University of Melbourne, Bachelor of Science, Diploma in Mathematical Sciences (Applied Maths)

2017-2018: Master of Science (Applied Mathematics)

2019-2024: PhD, MIT (Applied Mathematics)

Accepting students for VCE tutoring in Maths Methods, Specialist Maths and Physics! (and university maths/physics too) PM for more details

Alwin

  • Victorian
  • Forum Leader
  • ****
  • Posts: 838
  • Respect: +241
Re: VCE Physics Question Thread!
« Reply #333 on: November 02, 2013, 05:16:20 pm »
0
Magnetic field strength is directly proportional to the current.
What equation tells you that?
Biot-Savart law:

Oh wait, not part of course. My bad.

really? :OOOOOOOOOOOOO dang, they must have really cut down on the course. I swear Biot-Savart's Law was in it last year :o

JOKES, no one panic please, just nliu deciding to demonstrate his knowledge again =P
2012:  Methods [48] Physics [49]
2013:  English [40] (oops) Chemistry [46] Spesh [42] Indo SL [34] Uni Maths: Melb UMEP [4.5] Monash MUEP [just for a bit of fun]
2014:  BAeroEng/BComm

A pessimist says a glass is half empty, an optimist says a glass is half full.
An engineer says the glass has a safety factor of 2.0

lzxnl

  • Victorian
  • ATAR Notes Legend
  • *******
  • Posts: 3432
  • Respect: +215
Re: VCE Physics Question Thread!
« Reply #334 on: November 02, 2013, 07:36:29 pm »
0


So we can see that as the current increases, the magnetic field needs to increase or the time needs to be reduced.

This is only for an induced current, not for currents ttat create magnetic fields
2012
Mathematical Methods (50) Chinese SL (45~52)

2013
English Language (50) Chemistry (50) Specialist Mathematics (49~54.9) Physics (49) UMEP Physics (96%) ATAR 99.95

2014-2016: University of Melbourne, Bachelor of Science, Diploma in Mathematical Sciences (Applied Maths)

2017-2018: Master of Science (Applied Mathematics)

2019-2024: PhD, MIT (Applied Mathematics)

Accepting students for VCE tutoring in Maths Methods, Specialist Maths and Physics! (and university maths/physics too) PM for more details

joey7

  • Victorian
  • Forum Regular
  • **
  • Posts: 74
  • Respect: 0
  • School: Notre Dame College Shepparton
  • School Grad Year: 2013
Re: VCE Physics Question Thread!
« Reply #335 on: November 02, 2013, 07:50:09 pm »
0
Wait, so if a question talks about doubling the current how am I meant to know the effect on the field?
Also, I came across a question involving two parallel current carrying wires, where the distance between them was reduced by a factor of 2, In the answers it said field varies inversely with distance squared. Again how am I meant to know this?

Stevensmay

  • Guest
Re: VCE Physics Question Thread!
« Reply #336 on: November 02, 2013, 08:04:49 pm »
0
This is only for an induced current, not for currents ttat create magnetic fields

Skipped over part of the question, thanks.

lzxnl

  • Victorian
  • ATAR Notes Legend
  • *******
  • Posts: 3432
  • Respect: +215
Re: VCE Physics Question Thread!
« Reply #337 on: November 02, 2013, 11:18:00 pm »
+2
Wait, so if a question talks about doubling the current how am I meant to know the effect on the field?
Also, I came across a question involving two parallel current carrying wires, where the distance between them was reduced by a factor of 2, In the answers it said field varies inversely with distance squared. Again how am I meant to know this?

The awkward bit is...your answers are wrong. For parallel current carrying wires, the field strength is inversely proportional to the distance from the wire. I don't want to prove this here as 1. I would have to do some messy integration, or 2. I would have to introduce another physical law which would make no sense here.

You don't needed to know anything about magnetic field strengths in VCE, although maybe you need to know that increasing the distance generally decreases field strength (but not how).



That logic. We're taught that currents create a magnetic field, that increasing currents create larger magnetic fields and how magnetic forces operate, but we're not taught how to calculate magnetic field strengths. VCAA logic.
2012
Mathematical Methods (50) Chinese SL (45~52)

2013
English Language (50) Chemistry (50) Specialist Mathematics (49~54.9) Physics (49) UMEP Physics (96%) ATAR 99.95

2014-2016: University of Melbourne, Bachelor of Science, Diploma in Mathematical Sciences (Applied Maths)

2017-2018: Master of Science (Applied Mathematics)

2019-2024: PhD, MIT (Applied Mathematics)

Accepting students for VCE tutoring in Maths Methods, Specialist Maths and Physics! (and university maths/physics too) PM for more details

lolipopper

  • Victorian
  • Forum Obsessive
  • ***
  • Posts: 317
  • I'm making aaaalll kaaindzzz of gaains
  • Respect: -4
  • School: Lalor North Secondary College
  • School Grad Year: 2013
Re: VCE Physics Question Thread!
« Reply #338 on: November 02, 2013, 11:48:54 pm »
0
i'm not sure if i've asked this but are questions like Q2 of 2007 U3 motion still in course? i saw some on the older exams as well but didn't know how to do them well.
2014: Monash University, Law

lzxnl

  • Victorian
  • ATAR Notes Legend
  • *******
  • Posts: 3432
  • Respect: +215
Re: VCE Physics Question Thread!
« Reply #339 on: November 02, 2013, 11:56:44 pm »
0
It's a parabola; the particle's horizontal velocity component is constant but the vertical component increases in magnitude. Just draw something like that and I think you'll be fine.
2012
Mathematical Methods (50) Chinese SL (45~52)

2013
English Language (50) Chemistry (50) Specialist Mathematics (49~54.9) Physics (49) UMEP Physics (96%) ATAR 99.95

2014-2016: University of Melbourne, Bachelor of Science, Diploma in Mathematical Sciences (Applied Maths)

2017-2018: Master of Science (Applied Mathematics)

2019-2024: PhD, MIT (Applied Mathematics)

Accepting students for VCE tutoring in Maths Methods, Specialist Maths and Physics! (and university maths/physics too) PM for more details

lolipopper

  • Victorian
  • Forum Obsessive
  • ***
  • Posts: 317
  • I'm making aaaalll kaaindzzz of gaains
  • Respect: -4
  • School: Lalor North Secondary College
  • School Grad Year: 2013
Re: VCE Physics Question Thread!
« Reply #340 on: November 03, 2013, 12:49:56 pm »
0
It's a parabola; the particle's horizontal velocity component is constant but the vertical component increases in magnitude. Just draw something like that and I think you'll be fine.

thanks but are these types in the course, because it similar to relative motion using frames of references and stuff?
2014: Monash University, Law

Robert123

  • Victorian
  • Forum Obsessive
  • ***
  • Posts: 201
  • Respect: +5
  • School: Kyabram P-12 College
Re: VCE Physics Question Thread!
« Reply #341 on: November 03, 2013, 02:04:24 pm »
0
thanks but are these types in the course, because it similar to relative motion using frames of references and stuff?
I think it is sort of on the course since you have to be aware that for projectile motion, the horizontal component of the velocity is constant

sin0001

  • Victorian
  • Forum Obsessive
  • ***
  • Posts: 487
  • Respect: +1
  • School Grad Year: 2013
Re: VCE Physics Question Thread!
« Reply #342 on: November 03, 2013, 06:32:50 pm »
0
Do we have to substitute values in the formulas we use, for 2 marks in the exam? I recently checked a company's solutions and they allocated 1 mark for 'substituting values into correct formulas' and 1 mark for the correct answer, I thought that a mark would've been allocated just for writing down the correct formula...
ATAR: 99.00
Monash Commerce Scholars

Robert123

  • Victorian
  • Forum Obsessive
  • ***
  • Posts: 201
  • Respect: +5
  • School: Kyabram P-12 College
Re: VCE Physics Question Thread!
« Reply #343 on: November 03, 2013, 09:24:59 pm »
+1
Sin0001
Check out this link, it gives all the details regarding marking
http://www.vicphysics.org/documents/events/conf2009/Generalmarkingprinciples.doc

Henreezy

  • Victorian
  • Forum Regular
  • **
  • Posts: 66
  • Respect: +1
  • School: Narre Warren South P-12 College
  • School Grad Year: 2013
Re: VCE Physics Question Thread!
« Reply #344 on: November 09, 2013, 03:31:00 pm »
0
Question 4 b) VCAA 2012 Exam 1:
Is the tension force referred to as the 'gravitational force - upwards' due to the fact that the reaction is due to gravity? I don't know and that confuses me.

Action: Gravity 'pulling' it downwards
Reaction: Gravity 'pushing' it upwards (results in 'tension' from string?)

I'm so confused.
Last exam: 13th of November (Physics)
*[Sitting in Exam 1]* "If only I could remember the METHOD to answer this question" [crickets]
2012: Psychology
2013 Goals: 90+ ATAR
English (40+) | Methods CAS (37+) | Specialist (30+) | Physics (40+) |